David bought a bicycle for $75.00. If he sold it for 20% of the original price, how
much did he sell it for?

Answers

Answer 1

Answer:

$15

Step-by-step explanation:

75.00(0.20) = $15


Related Questions

In 2005 the U.S. Census Bureau reported that 68.9% of American families owned their homes. Census data reveal that the ownership rate in one small city is much lower. The city council is debating a plan to offer tax breaks to first-time home buyers in order to encourage people to become homeowners. They decide to adopt the plan on a 2-year trial basis and use the data they collect to make a decision about continuing the tax breaks. Since this plan costs the city tax revenues, they will continue to use it only if there is strong evidence that the rate of home ownership is increasing. a) In words, what will their hypotheses be

Answers

Answer:

H0 : Number of homeowners stays the same

H1: Number of homeowners increases.

Step-by-step explanation:

The hypothesis is to test if the level of home ownership rises or doesn't.

For this type of experiment,

The null hypothesis is the current level of home ownership.

The alternative hypothesis which is usually the opposite of the null is meant to challenge the claim of the Null. Hence, the null hypothesis will support the claim or hypothesis of the study, which is that the number of homeowners increases, hence, does not remain the same.

Therefore,

H0 : Number of homeowners stays the same

H1: Number of homeowners increases.

1 Are these similar triangles? ​

Answers

Answer:

NOPE

Step-by-step explanation:

Find the length of side x in the simplest radical form with a rational denominator.

Answers

Answer:

[tex]\sqrt{2}[/tex]

Step-by-step explanation:

We can use either angle, but I'm going to use the one on the bottom. So, in order to find x, we need to use tangent. One side we know is the adjacent, and the side we don't know is the opposite, therefore we need tangent. Here's the equation:

[tex]tan(45)=\frac{x}{\sqrt{2}}[/tex]

Obviously, we can't have a root in our denominator, so we need to get rid of it somehow. Here's how:

We multiply the denominator of the fraction by [tex]\sqrt{2}[/tex]. [tex]\sqrt{2}[/tex] multiplied by itself is simply 2. Try it! We also want to multiply the numerator by [tex]\sqrt{2}[/tex], but there isn't really a number we can use with that, so we'll just add it to the side. The equation you have now is:

[tex]tan(45)=\frac{x\sqrt{2} }{2}[/tex]

Let's try to work this out now. Since the denominator is 2, we have to multiply both sides by it to find x.

[tex]\frac{x\sqrt{2} }{2*2} =x\sqrt{2}[/tex]

[tex]tan(45)*2=2[/tex]

We can plug 2 in for the x in the numerator now:

[tex]\frac{2\sqrt{2} }{2}[/tex]

2 and 2 cancel out, so you get 1 in both the numerator and denominator. That's how we get our answer of [tex]x=\sqrt{2}[/tex]

Also, because this is a 45-45-90 triangle, you don't really have to do all that work. If it's a 45-45-90 triangle, both legs should be the same length. :)

whats the volume (5 points)​

Answers

Answer is: 20 because, u multiple 5 times 2 then, 10 times 2 which is 20 :D

Solve 81^x=27^x +2
x=1
x=2
x=5
x=6

Answers

Answer:

x=6

Step-by-step explanation:

Answer:

x=6

Step-by-step explanation:

[tex]{3}^{4x} = {3}^{3( x+ 2)} [/tex]

4x = 3(x + 2)

4x = 3x + 6

4x - 3x = 6

x = 6

.What is the perimeter of the figure below?*
2 in
9 in
2 in
11 in

Please help!

Answers

Answer:

I think 24

Step-by-step explanation:

11+9=20+2+2=24

i need help with graph pleasee​

Answers

Answer:

I drew the graph

I hope this helps you

do u need it still? i can help if not

Choose the ordered pair(s) that are solutions to the system. (There may be one or many correct options).

-3x-y>-4
2x-5y<-2

Answer Choices:
a. (5,5)
b. (2,6)
c. (-4,1)
d. (6,5)
e. (-5,0)
f. none of the above

Answers

Answer: C,E

Step-by-step explanation: This is because when we plug in 5 for x and  for y in the first inequality we get.

-3(5)-5>-4

Which equals -20>-4 and thats false so thats not it.

Then we got to (2,6) and we plug 2 for x and 6 for y and get:

-3(2)-6>-4

Which is -12>-4 which is false so thats also not it.

Then we got to C and we plug in -4 for x and 1 for y and we get:

-3(-4)-1>-4 and we get 12-1>-4

This is correct so we plug it in to the next inequality and get:

2(-4)-1<-2 and we get -9<-2 which is true

So C is an answer and we go on to d and e

For d we plug in 6 for x and 5 for y and we get:

-3(6)-5>-4 which is -23>-4 which is false so its not d

And then we go to e which is -5 for x and 0 for y and we plug in and we get:

-3(-5)-0>-4 This is 15>-4 which is true so we go to the next inequality and we plug in -5 for x and 0 for y and we get:

2(-5)-5(0)<-2 This is -10<-2 which is true so e is the other answer so the answers are

(C,E)

lec types 3/5 of a paragraph in 2/3 minute. If he continues at the same rate, what fraction of a paragraph can Alec complete in 1 minute?

Answers

Answer:

ik you in MS 118 but still the answer is 9/10

Step-by-step explanation:

multiply by the reciprocal 3/2.

3/5x3/2=9/10

What is the rate of change in the equation?

Answers

Answer:

-5

Step-by-step explanation:

The rate of change is normally the number not attached to X so in this equation it is-5.

The solution to 8x = 32 is x = ___.

a.3

b.4

c.5

d.6

Answers

Answer:

b.4

Step-by-step explanation:

8x=32

x = 32 / 8

x = 4

PLEASE HELP I HAVE 30 MINUTES UNTIL CLASS ENDS PLEASE HELP

Answers

Answer:

12 stamps

Step-by-step explanation:

6 divided by 0.50 = 12

Answer: it’s 12

Step-by-step explanation: all I did was multiplied 6 by .50

Hope this helps!

Two angles are complementary. One angle is 18 degrees more than the other. What are the measures of the two angles?​

Answers

Answer:

Step-by-step explanation:

x+y = 90°

x = y+18°

(y+18°) + y = 90°

2y = 72°

y = 36°

x = y+18° = 54°

7.
Wendy is paid $12 per hour and plans to work between 30 and 35 hours per week. Identify the independent and dependent quantity in the situation. Find reasonable domain and range values.


A. weekly pay; hours worked; $360 to $420; 30 to 35 hours

B. hours worked; weekly pay; 30 to 35 hours; $360 to $420

C. weekly pay; hours worked; 30 to 35 hours; $360 to $420

D. hours worked; weekly pay; 30 to 35 hours; $0 to $420

Answers

Answer:   I'm gonna go with B. hours worked; weekly pay; 30 to 35 hours; $360 to $420, because I had something similar to this

Wendy is paid $9 per hour and plans to work between 20 and 25 hours per week. Identify the independent and dependent quantity in the situation. Find reasonable domain and range values.

 

weekly pay; hours worked; 20 to 25 hours; $180 to $225

 

hours worked; weekly pay; 20 to 25 hours; $180 to $225

 

weekly pay; hours worked; $180 to $225; 20 to 25 hours

 

hours worked; weekly pay; 20 to 25 hours; $9 to $225

Step-by-step explanation:

The independent quantity is her pay and the dependent quantity is the hours she works in a week.

I believe the range would be 30 to 35 hours and the domain would be $360 to $450.

...................................................................................................................................................

Answer:

Part a) The independent quantity is the variable x (the number of hours) and the dependent quantity is the variable y (total paid)

Part b) Domain  

Part c) Range  

Step-by-step explanation:

Let

x------> the number of hours

y-----> total paid

The linear equation that represent this situation is

so

Part a)

The independent quantity is the variable x (the number of hours)

The dependent quantity is the variable y (total paid)

Part b) Find the domain

The domain is the interval-------->  

All real numbers greater than or equal to  hours and less than or equal to  hours

Part c) Find the range

For  

find the value of y

For  

find the value of y

The range is the interval------->  

All real numbers greater than or equal to  and less than or equal to  

...............................................................................................................................................

The dependent quantity would be the $12 per hour, while the independent quantities are 30 hrs/ week to 35 hours/ week.

The $12 per hour would become $360/week to $420/week.

The domain of the problem, with the unit of hrs/week, would be :30,31,32,33,34,35.

The Range of the problem, with the unit of $/week, would be : 360, 372, 384, 396, 408, 420.

...............................................................................................................................................

PLSS HELPP!!! I WILL BE GIVING BRAINLIEST AND POINTS:(!!

Answers

Answer:

A≈1628.55

Step-by-step explanation:

A=(6ah)+3sqr3a^2=6·8·27+3·sqr3·8^2≈1628.55376

solve all of these ?

Answers

21 Answer:

[tex] \frac{3}{a} [/tex]

22 Answer:

[tex] \frac{1}{ {b}^{3} } [/tex]

23 Answer:

[tex] \frac{ {4a}^{2}c^{3} }{ {b}^{2} } [/tex]

24 Answer:

[tex]9c ^{4} [/tex]

25 Answer:

[tex] - \frac{1}{ {a}^{2} } [/tex]

26 Answer:

[tex] \frac{1}{ {c}^{2} } [/tex]

William bought some cashews and walnuts. Spending the same amount of money as William, his twin Jillian bought twice as many cashews and 10 less walnuts. If the cost of one walnut is 2 more than the cost of cashew, then what is the minimum possible number of cashews bought by the twins together?

Answers

Answer: 33

Step-by-step explanation:

Given

The cost of one walnut is 2 more than the cost of a cashew

Jillian bought twice the amount of cashew as William and 10 fewer walnuts

Suppose the price of cashew and walnuts be x and x+2

Assume William bought a cashew and b walnuts

Money spent by Willian

[tex]\Rightarrow W=a(x)+b(x+2)[/tex]

Money spent by Jillian

[tex]\Rightarrow J=2a(x)+(b-10)(x+2)[/tex]

They spent equal money i.e.

[tex]\Rightarrow ax+bx+2b=2ax+bx+2b-10x-20\\\Rightarrow ax-10x-20=0\\\Rightarrow (a-10)x=20\\\text{Here product of two number is 20 , so both must be greater than 0}\\\therefore a-10>0\\\Rightarrow a>10\\\text{i.e. minimum value of a is 11}[/tex]

The amount of cashew both twins bought is a+2a=3a

[tex]\Rightarrow 3a=3\times 11=33[/tex]

A person invests 3500 dollars in a bank. The bank pays 6.25% interest compounded annually. To the nearest tenth of a year, how long must the person leave the money in the bank until it reaches 11100 dollars?

Answers

18.328 or 18.3 years

The duration of time period required is 2.4 years.

What is a compound interest?

Compound interest simply means that the interest associated with a bank account, loan, or investment increases exponentially over time.

Given that, A person invests 3500 dollars in a bank. The bank pays 6.25% interest compounded annually.

A = P(1+r/100)ⁿ

n = time

A = amount

P = principal

11100 = 3500(1+0.0625)ⁿ

3.171 = (1.0625)ⁿ

Taking log to both sides,

log(3.171) = nlog(1.0625)

n = 2.40

Hence, the required time is 2.4 years.

For more references on compound interest, click;

https://brainly.com/question/14295570

#SPJ2

A dump truck delivers dirt for $6 per cubit foot. The dimensions are 4 3/4 ft by 8 ft by 17 ft. How much does a full load of dirt cost

Answers

Answer:

None. There can't be any dirt in it because it's a hole.

Step-by-step explanation:

Which of the following equations is equivalent to y-3 = -2(x-6)?

а. y=-5x-9

b. y = -2x 3

с. y= -5x - 15

d. y = -2x + 9

Answers

Answer:

Step-by-step explanation:

Which of the following equations is equivalent to y-3=-2(x-6): y+3x=6 ... Move the +3*x to the right side by subtracting 3*x from (or adding negative 3*x to) ... And in addition, we know that the line graph crosses the y-axis where y equals y=-2x3.

What is the solution to this equation?

2 x + 10 = 28



Record your answer in the space provided. Be sure to use the correct place value.

Answers

Answer:

[tex]2x + 10 = 28 \\ 2x = 28 - 10 \\ 2x = 18 \\ 2x \div 2 = 18 \div 2 \\ x = 14[/tex]

Determine the intercepts of the line.
y = -32 + 12
Y-
X-

Answers

Answer:

y = − 20

x = none

Step-by-step explanation:

There is no X intercepts

-32 + 12 = 20

A bullet fired from a gun has a muzzle velocity of 3510 km per hour how long dies it take to travel 100m

Answers

Answer:

A bullet fired from a gun has a muzzle velocity of 3510 km per hour how long dies it take to travel 100m

Step-by-step explanation:

Given :

Velocity of bullet = 3510 km/hr

Distance traveled = 100m

Converting distance in meter to kilometer :

(100 / 1000) km = 0.1 km

Recall :

Speed = Distance /time

Time taken = Distance / speed

Therefore,

Time taken = Distance / speed

Time taken = 0.1 km / 3510 km/hr

Time = 2.849 * 10^-5 hour

Someone help please

Answers

Answer:

-2x2 y1/3

Step-by-step explanation:

Find the value or measure. Round answers to the nearest tenth, as needed.

Answers

Answer:

26

Step-by-step explanation:

Solve 9x-1 = 41-5x

9x-1 = 41-5x

   +1   +1

9x = 42-5x

+5x      +5x

14x = 42

/14     /14

x = 3

Plug in X for EF

41-5(3)

41-15

=26

Can someone help me with this ASAP please I’m being timed !

Answers

Answer:

b. y = 5/3x

Step-by-step explanation:

i hope this helps :)

Find the equation of the line below.
(3,1)

Answers

Answer:

B

Step-by-step explanation:

the answer is B.the equation of the line

The base of a triangle is 2 more than three times the side lengths (x). If the
perimeter is equal to 97, then find the base .

Answers

Answer: 15

Step-by-step explanation:

Base is y
Side lengths x
y+2=3•x
P=2x+y
2x+y=97
2x=97-y
x=97-y/2 plug that into
3x=y+2
3(97-y/2)=y+2 multiply by 2
3(97-y)=2(y+2)
292-3y=2y+4
-3y-2y=4-292 multiply by (-1)
5y=288
y=288/5
y=57.6

3x=y+2
3x=57.6+2
3x=59.6
x=19.86

97=2•19.86+57.6
97=39.72+57.6
97=97.32
This is the best I came up with.

Good luck

At the beginning of March, a store bought a rake at a cost of 9$ and marked it up 130%. At the end of the month, the rake had not sold, so the store marked it down 50%. What is the discounted price?

Answers

Answer:

the new discounted price is $5.85

Step-by-step explanation:

First convert 130% into a decimal: 130 ÷ 100 = 1.3

Next multiply 1.3 by 9: 9 x 1.3 = 11.7

Then convert 50% into a decimal: 50 ÷ 100 = 0.5

Last multiply 11.7 by 0.5: 11.7 x 0.5 = 5.85

Solve for x and y simultaneously

Pls help me ​

Answers

I don't think its possible to do this... you have to have 2 equations

Here is it Y= 2/5x + 6/5, you just need to write it in the explicit form and then simplify it
Other Questions
Which of the following is TRUE about factories during the industrial revolutionA) Workers in factories only worked 8 hours a day 5 days a week.B) Factories paid their workers very high salaries.C) Factories were very clean and safe during the industrial revolution.D) Factories were dangerous and workers worked long hours for very little pay.no links/files that dont work cause it wont let me go on there Question 5Ulysses gave Polyphemus the wine in order to get him drunk and sleepy.TrueFalse Drag the tiles to the correct boxes to complete the pairs. Not all tiles will be used. Match the estimated value of each expression with its position on the number line. which is grater .856 or .8561 Given : 4.50 molteHow many moles of ammonia would you get if 4.50 moles of hydrogen gasreacted Moijo is an Alaska Native who has resided in the region for many years. His community feels very strongly about its traditional subsistence rights. Which activity shows an example of these subsistence rights?A. huntingB. agricultureC. tradeD. mining What fraction of the catapults flung a marshmallow 30 inches or less? A 20.0 mL of 0.27 M solution of the salt NaA has a pH of 8.70.a) Calculate the pk, value of the acid HA.(Correct to 3 sig. fig)b) Calculate the pH of a solution containing 0.3 M HA and 0.1 M NaA.(Correct to 3 sig. fig)Thanks!! Which probability indicates that an event will definitely occur? 1/4 1/2 3/4 1 Please Help!!!! need it just right now Can you please help me find the volume of this shape ? what president expanded the conflict in 1964? Part BWhich sentence is most effective in supporting thisdescription?A. Della finished her cry and attended to hercheeks with the powder rag.B. Tomorrow would be Christmas Day, andshe had only $1.87 with which to buy Jima present.thyC. Expenses had been greater than she hadcalculated.D. Many a happy hour she had spentplanning for something nice for him. 72 is what percent of 180 expand: 0.25(xy+4z+5) Solve the inequality:2x+1016Select one:x13x13x3x3 Had the most railroad milage and train enginesA. NorthB. South A sample of neon gas is at a temperature of 60C and has a volume of 300 cm. What is the volume of the gas if it is heated to 100C at a constant pressure? What is the difference between a short term savings account and a long term savings account? help me this is hard